Pythagorean triple











up vote
0
down vote

favorite













  1. Show that neither $1$ not $2$ can appear in any Pythagorean triple, but that every integer $kgeq3$ can appear.


  2. Prove that for each integer $k$ there are only finitely many Pythagorean triple containing $k$.



Help me to prove this...
Thank you.










share|cite|improve this question




















  • 1




    Welcome to MSE. Please show any working out you have done. That way, more people will be willing to help you.
    – Kyky
    Nov 17 at 1:55












  • I have no idea......
    – Nithish Kumar R
    Nov 17 at 1:59










  • If $1$ appeared in a Pythagorean triple, we would have $c^2-b^2=1$. It means that there should be two consecutive integers that are also perfect squares. Since there are no consecutive squares, $1$ cannot appear in a Pythagorean triple. You can use the same argument with $2$ and, with some modifications, with $3$ and other numbers.
    – rafa11111
    Nov 17 at 2:06

















up vote
0
down vote

favorite













  1. Show that neither $1$ not $2$ can appear in any Pythagorean triple, but that every integer $kgeq3$ can appear.


  2. Prove that for each integer $k$ there are only finitely many Pythagorean triple containing $k$.



Help me to prove this...
Thank you.










share|cite|improve this question




















  • 1




    Welcome to MSE. Please show any working out you have done. That way, more people will be willing to help you.
    – Kyky
    Nov 17 at 1:55












  • I have no idea......
    – Nithish Kumar R
    Nov 17 at 1:59










  • If $1$ appeared in a Pythagorean triple, we would have $c^2-b^2=1$. It means that there should be two consecutive integers that are also perfect squares. Since there are no consecutive squares, $1$ cannot appear in a Pythagorean triple. You can use the same argument with $2$ and, with some modifications, with $3$ and other numbers.
    – rafa11111
    Nov 17 at 2:06















up vote
0
down vote

favorite









up vote
0
down vote

favorite












  1. Show that neither $1$ not $2$ can appear in any Pythagorean triple, but that every integer $kgeq3$ can appear.


  2. Prove that for each integer $k$ there are only finitely many Pythagorean triple containing $k$.



Help me to prove this...
Thank you.










share|cite|improve this question
















  1. Show that neither $1$ not $2$ can appear in any Pythagorean triple, but that every integer $kgeq3$ can appear.


  2. Prove that for each integer $k$ there are only finitely many Pythagorean triple containing $k$.



Help me to prove this...
Thank you.







number-theory pythagorean-triples






share|cite|improve this question















share|cite|improve this question













share|cite|improve this question




share|cite|improve this question








edited Nov 17 at 1:56









Tianlalu

2,8811935




2,8811935










asked Nov 17 at 1:52









Nithish Kumar R

91




91








  • 1




    Welcome to MSE. Please show any working out you have done. That way, more people will be willing to help you.
    – Kyky
    Nov 17 at 1:55












  • I have no idea......
    – Nithish Kumar R
    Nov 17 at 1:59










  • If $1$ appeared in a Pythagorean triple, we would have $c^2-b^2=1$. It means that there should be two consecutive integers that are also perfect squares. Since there are no consecutive squares, $1$ cannot appear in a Pythagorean triple. You can use the same argument with $2$ and, with some modifications, with $3$ and other numbers.
    – rafa11111
    Nov 17 at 2:06
















  • 1




    Welcome to MSE. Please show any working out you have done. That way, more people will be willing to help you.
    – Kyky
    Nov 17 at 1:55












  • I have no idea......
    – Nithish Kumar R
    Nov 17 at 1:59










  • If $1$ appeared in a Pythagorean triple, we would have $c^2-b^2=1$. It means that there should be two consecutive integers that are also perfect squares. Since there are no consecutive squares, $1$ cannot appear in a Pythagorean triple. You can use the same argument with $2$ and, with some modifications, with $3$ and other numbers.
    – rafa11111
    Nov 17 at 2:06










1




1




Welcome to MSE. Please show any working out you have done. That way, more people will be willing to help you.
– Kyky
Nov 17 at 1:55






Welcome to MSE. Please show any working out you have done. That way, more people will be willing to help you.
– Kyky
Nov 17 at 1:55














I have no idea......
– Nithish Kumar R
Nov 17 at 1:59




I have no idea......
– Nithish Kumar R
Nov 17 at 1:59












If $1$ appeared in a Pythagorean triple, we would have $c^2-b^2=1$. It means that there should be two consecutive integers that are also perfect squares. Since there are no consecutive squares, $1$ cannot appear in a Pythagorean triple. You can use the same argument with $2$ and, with some modifications, with $3$ and other numbers.
– rafa11111
Nov 17 at 2:06






If $1$ appeared in a Pythagorean triple, we would have $c^2-b^2=1$. It means that there should be two consecutive integers that are also perfect squares. Since there are no consecutive squares, $1$ cannot appear in a Pythagorean triple. You can use the same argument with $2$ and, with some modifications, with $3$ and other numbers.
– rafa11111
Nov 17 at 2:06












3 Answers
3






active

oldest

votes

















up vote
2
down vote













$1^2$ and $2^2$ are not sums of two positive integers squares so for any Pythagorean
triple $(x, y, z)$ we must have $z ge 3$.



Now $x^2 < x^2 + 1 < (x + 1)^2$ for any $x in N$, so $x^2 + 1^2$ is not a perfect square which gives $1 in { x, y, z}$. i.e. $x ne 1, y ne 1$, and so $x ge 2, y ge 2$. If $y = 2$, then $x^2 < x^2 +y^2 = x^2 + 4 < (x+ 1)^2$
, since $x ge 2$, and thus $x^2 +y^2$ is not a perfect square, so $y ne 2$. Similarly $x ne 2$. Thus ${x, y, z} ⊆ {3, 4, 5, 6, · · · }$.



Let $k ∈ N, k ge 3$ any. If $k$ is odd, then $k^2 = 2l + 1$ with $l ge 4$ and $l^2 + k^2 = (l + 1)^2$;



if $k$ is even, then $k^2 = 4l$ with $l ge 4$ and $(l − 1)^2 + k^2 = (l + 1)^2$



Thus for any integer $k ge 3$, we have a Pythagorean triple $(x, y, z)$ such that $k ∈ {x, y, z}$






share|cite|improve this answer





















  • I think you mean "are not the difference of two positive integer squares" in the first phrase.
    – rafa11111
    Nov 17 at 2:18


















up vote
0
down vote













Since the formula $a^2+b^2=c^2$ can be changed into $a^2=c^2-b^2$, $a^2$ needs to mean the difference between $2$ squares. You will find that $(a+1)^2-a^2$ is equal to $a^2-2a+1-a^2=2a+1$ and $(a+2)^2-a^2=a^2+4a+4-a^2=4a+4$. The former must be odd and the latter must be even. These are obtained by unpacking the bracket and simplifying. This is the maximum values of $c^2$ and $b^2$ of which $a^2=c^2-b^2$ stand true as if there were any larger then $c^2$ and $b^2$ are fractional. For $1$, the largest values of $c^2$ and $b^2$ is $0^2$ and $1^2$ respectively, but $0^2$ can't be part of a side. Again, $2^2$ is the difference between $0^2$ and $2^2$, but $0^2$ can't be used.



On to the second question. We have already proven that the above formulae show that there is a limit as to the values of $b^2$ and $c^2$, and if there are infinite possible values of $b^2$ and $c^2$ then some of the must be non-integer. So by proof of contradiction for any real integer $k$, there can only be a finite amount of Pythagorean triples.






share|cite|improve this answer




























    up vote
    0
    down vote













    We have $(a^2+b^2)^2=(a^2-b^2)^2+(2ab)^2.$



    (1). The sequence $(2^2-1^2, 3^2-2^2, 4^2-3^2,...)$ of differences of successive squares is the sequence $(3,5,7,...)$ of odd numbers $>1$. So if $m $ is odd and $mgeq 3$ then we can find $b$ such that $(b+1)^2-b^2=m.$ That is, $2b+1=m.$ So let $b=(m-1)/2$ and $a=b+1.$ Then we have $$(a^2+b^2)^2=(a^2-b^2)+(2ab)^2=m^2+(2ab)^2.$$This does not work for $m=1$ as the member $2ab$ of the triplet would be $0.$



    (2). If $m$ is even and $mgeq 4$, let $a=m/2$ and $b=1.$ Then we have $$(a^2+b^2)^2 =(a^2-b^2)^2+(2ab)^2=(a^2-b^2)+m^2.$$This does not work for $m=2$ as the member $a^2-b^2$ of the triplet would be $0$.



    (3). If $x,y,z$ are positive integers with $x^2+y^2=z^2$ then



    (3-i). It is easy to confirm that $1ne zne 2.$



    (3-ii). We have $y<z$ so $ yleq z-1$ so $x^2=z^2-y^2geq z^2-(z-1)^2=2z-1geq 5$, so $x>2.$ Interchanging $x,y$ in this, we also get $y^2geq 5$ so $y>2$.



    BTW. The general formula for ALL Pyth. triplets is ${k(a^2-b^2),,2kab,, k(a^2+b^2)}$ for $a,b,kin Bbb N$ with $a>b$.






    share|cite|improve this answer





















      Your Answer





      StackExchange.ifUsing("editor", function () {
      return StackExchange.using("mathjaxEditing", function () {
      StackExchange.MarkdownEditor.creationCallbacks.add(function (editor, postfix) {
      StackExchange.mathjaxEditing.prepareWmdForMathJax(editor, postfix, [["$", "$"], ["\\(","\\)"]]);
      });
      });
      }, "mathjax-editing");

      StackExchange.ready(function() {
      var channelOptions = {
      tags: "".split(" "),
      id: "69"
      };
      initTagRenderer("".split(" "), "".split(" "), channelOptions);

      StackExchange.using("externalEditor", function() {
      // Have to fire editor after snippets, if snippets enabled
      if (StackExchange.settings.snippets.snippetsEnabled) {
      StackExchange.using("snippets", function() {
      createEditor();
      });
      }
      else {
      createEditor();
      }
      });

      function createEditor() {
      StackExchange.prepareEditor({
      heartbeatType: 'answer',
      convertImagesToLinks: true,
      noModals: true,
      showLowRepImageUploadWarning: true,
      reputationToPostImages: 10,
      bindNavPrevention: true,
      postfix: "",
      imageUploader: {
      brandingHtml: "Powered by u003ca class="icon-imgur-white" href="https://imgur.com/"u003eu003c/au003e",
      contentPolicyHtml: "User contributions licensed under u003ca href="https://creativecommons.org/licenses/by-sa/3.0/"u003ecc by-sa 3.0 with attribution requiredu003c/au003e u003ca href="https://stackoverflow.com/legal/content-policy"u003e(content policy)u003c/au003e",
      allowUrls: true
      },
      noCode: true, onDemand: true,
      discardSelector: ".discard-answer"
      ,immediatelyShowMarkdownHelp:true
      });


      }
      });














      draft saved

      draft discarded


















      StackExchange.ready(
      function () {
      StackExchange.openid.initPostLogin('.new-post-login', 'https%3a%2f%2fmath.stackexchange.com%2fquestions%2f3001873%2fpythagorean-triple%23new-answer', 'question_page');
      }
      );

      Post as a guest















      Required, but never shown

























      3 Answers
      3






      active

      oldest

      votes








      3 Answers
      3






      active

      oldest

      votes









      active

      oldest

      votes






      active

      oldest

      votes








      up vote
      2
      down vote













      $1^2$ and $2^2$ are not sums of two positive integers squares so for any Pythagorean
      triple $(x, y, z)$ we must have $z ge 3$.



      Now $x^2 < x^2 + 1 < (x + 1)^2$ for any $x in N$, so $x^2 + 1^2$ is not a perfect square which gives $1 in { x, y, z}$. i.e. $x ne 1, y ne 1$, and so $x ge 2, y ge 2$. If $y = 2$, then $x^2 < x^2 +y^2 = x^2 + 4 < (x+ 1)^2$
      , since $x ge 2$, and thus $x^2 +y^2$ is not a perfect square, so $y ne 2$. Similarly $x ne 2$. Thus ${x, y, z} ⊆ {3, 4, 5, 6, · · · }$.



      Let $k ∈ N, k ge 3$ any. If $k$ is odd, then $k^2 = 2l + 1$ with $l ge 4$ and $l^2 + k^2 = (l + 1)^2$;



      if $k$ is even, then $k^2 = 4l$ with $l ge 4$ and $(l − 1)^2 + k^2 = (l + 1)^2$



      Thus for any integer $k ge 3$, we have a Pythagorean triple $(x, y, z)$ such that $k ∈ {x, y, z}$






      share|cite|improve this answer





















      • I think you mean "are not the difference of two positive integer squares" in the first phrase.
        – rafa11111
        Nov 17 at 2:18















      up vote
      2
      down vote













      $1^2$ and $2^2$ are not sums of two positive integers squares so for any Pythagorean
      triple $(x, y, z)$ we must have $z ge 3$.



      Now $x^2 < x^2 + 1 < (x + 1)^2$ for any $x in N$, so $x^2 + 1^2$ is not a perfect square which gives $1 in { x, y, z}$. i.e. $x ne 1, y ne 1$, and so $x ge 2, y ge 2$. If $y = 2$, then $x^2 < x^2 +y^2 = x^2 + 4 < (x+ 1)^2$
      , since $x ge 2$, and thus $x^2 +y^2$ is not a perfect square, so $y ne 2$. Similarly $x ne 2$. Thus ${x, y, z} ⊆ {3, 4, 5, 6, · · · }$.



      Let $k ∈ N, k ge 3$ any. If $k$ is odd, then $k^2 = 2l + 1$ with $l ge 4$ and $l^2 + k^2 = (l + 1)^2$;



      if $k$ is even, then $k^2 = 4l$ with $l ge 4$ and $(l − 1)^2 + k^2 = (l + 1)^2$



      Thus for any integer $k ge 3$, we have a Pythagorean triple $(x, y, z)$ such that $k ∈ {x, y, z}$






      share|cite|improve this answer





















      • I think you mean "are not the difference of two positive integer squares" in the first phrase.
        – rafa11111
        Nov 17 at 2:18













      up vote
      2
      down vote










      up vote
      2
      down vote









      $1^2$ and $2^2$ are not sums of two positive integers squares so for any Pythagorean
      triple $(x, y, z)$ we must have $z ge 3$.



      Now $x^2 < x^2 + 1 < (x + 1)^2$ for any $x in N$, so $x^2 + 1^2$ is not a perfect square which gives $1 in { x, y, z}$. i.e. $x ne 1, y ne 1$, and so $x ge 2, y ge 2$. If $y = 2$, then $x^2 < x^2 +y^2 = x^2 + 4 < (x+ 1)^2$
      , since $x ge 2$, and thus $x^2 +y^2$ is not a perfect square, so $y ne 2$. Similarly $x ne 2$. Thus ${x, y, z} ⊆ {3, 4, 5, 6, · · · }$.



      Let $k ∈ N, k ge 3$ any. If $k$ is odd, then $k^2 = 2l + 1$ with $l ge 4$ and $l^2 + k^2 = (l + 1)^2$;



      if $k$ is even, then $k^2 = 4l$ with $l ge 4$ and $(l − 1)^2 + k^2 = (l + 1)^2$



      Thus for any integer $k ge 3$, we have a Pythagorean triple $(x, y, z)$ such that $k ∈ {x, y, z}$






      share|cite|improve this answer












      $1^2$ and $2^2$ are not sums of two positive integers squares so for any Pythagorean
      triple $(x, y, z)$ we must have $z ge 3$.



      Now $x^2 < x^2 + 1 < (x + 1)^2$ for any $x in N$, so $x^2 + 1^2$ is not a perfect square which gives $1 in { x, y, z}$. i.e. $x ne 1, y ne 1$, and so $x ge 2, y ge 2$. If $y = 2$, then $x^2 < x^2 +y^2 = x^2 + 4 < (x+ 1)^2$
      , since $x ge 2$, and thus $x^2 +y^2$ is not a perfect square, so $y ne 2$. Similarly $x ne 2$. Thus ${x, y, z} ⊆ {3, 4, 5, 6, · · · }$.



      Let $k ∈ N, k ge 3$ any. If $k$ is odd, then $k^2 = 2l + 1$ with $l ge 4$ and $l^2 + k^2 = (l + 1)^2$;



      if $k$ is even, then $k^2 = 4l$ with $l ge 4$ and $(l − 1)^2 + k^2 = (l + 1)^2$



      Thus for any integer $k ge 3$, we have a Pythagorean triple $(x, y, z)$ such that $k ∈ {x, y, z}$







      share|cite|improve this answer












      share|cite|improve this answer



      share|cite|improve this answer










      answered Nov 17 at 2:06









      Key Flex

      7,03931229




      7,03931229












      • I think you mean "are not the difference of two positive integer squares" in the first phrase.
        – rafa11111
        Nov 17 at 2:18


















      • I think you mean "are not the difference of two positive integer squares" in the first phrase.
        – rafa11111
        Nov 17 at 2:18
















      I think you mean "are not the difference of two positive integer squares" in the first phrase.
      – rafa11111
      Nov 17 at 2:18




      I think you mean "are not the difference of two positive integer squares" in the first phrase.
      – rafa11111
      Nov 17 at 2:18










      up vote
      0
      down vote













      Since the formula $a^2+b^2=c^2$ can be changed into $a^2=c^2-b^2$, $a^2$ needs to mean the difference between $2$ squares. You will find that $(a+1)^2-a^2$ is equal to $a^2-2a+1-a^2=2a+1$ and $(a+2)^2-a^2=a^2+4a+4-a^2=4a+4$. The former must be odd and the latter must be even. These are obtained by unpacking the bracket and simplifying. This is the maximum values of $c^2$ and $b^2$ of which $a^2=c^2-b^2$ stand true as if there were any larger then $c^2$ and $b^2$ are fractional. For $1$, the largest values of $c^2$ and $b^2$ is $0^2$ and $1^2$ respectively, but $0^2$ can't be part of a side. Again, $2^2$ is the difference between $0^2$ and $2^2$, but $0^2$ can't be used.



      On to the second question. We have already proven that the above formulae show that there is a limit as to the values of $b^2$ and $c^2$, and if there are infinite possible values of $b^2$ and $c^2$ then some of the must be non-integer. So by proof of contradiction for any real integer $k$, there can only be a finite amount of Pythagorean triples.






      share|cite|improve this answer

























        up vote
        0
        down vote













        Since the formula $a^2+b^2=c^2$ can be changed into $a^2=c^2-b^2$, $a^2$ needs to mean the difference between $2$ squares. You will find that $(a+1)^2-a^2$ is equal to $a^2-2a+1-a^2=2a+1$ and $(a+2)^2-a^2=a^2+4a+4-a^2=4a+4$. The former must be odd and the latter must be even. These are obtained by unpacking the bracket and simplifying. This is the maximum values of $c^2$ and $b^2$ of which $a^2=c^2-b^2$ stand true as if there were any larger then $c^2$ and $b^2$ are fractional. For $1$, the largest values of $c^2$ and $b^2$ is $0^2$ and $1^2$ respectively, but $0^2$ can't be part of a side. Again, $2^2$ is the difference between $0^2$ and $2^2$, but $0^2$ can't be used.



        On to the second question. We have already proven that the above formulae show that there is a limit as to the values of $b^2$ and $c^2$, and if there are infinite possible values of $b^2$ and $c^2$ then some of the must be non-integer. So by proof of contradiction for any real integer $k$, there can only be a finite amount of Pythagorean triples.






        share|cite|improve this answer























          up vote
          0
          down vote










          up vote
          0
          down vote









          Since the formula $a^2+b^2=c^2$ can be changed into $a^2=c^2-b^2$, $a^2$ needs to mean the difference between $2$ squares. You will find that $(a+1)^2-a^2$ is equal to $a^2-2a+1-a^2=2a+1$ and $(a+2)^2-a^2=a^2+4a+4-a^2=4a+4$. The former must be odd and the latter must be even. These are obtained by unpacking the bracket and simplifying. This is the maximum values of $c^2$ and $b^2$ of which $a^2=c^2-b^2$ stand true as if there were any larger then $c^2$ and $b^2$ are fractional. For $1$, the largest values of $c^2$ and $b^2$ is $0^2$ and $1^2$ respectively, but $0^2$ can't be part of a side. Again, $2^2$ is the difference between $0^2$ and $2^2$, but $0^2$ can't be used.



          On to the second question. We have already proven that the above formulae show that there is a limit as to the values of $b^2$ and $c^2$, and if there are infinite possible values of $b^2$ and $c^2$ then some of the must be non-integer. So by proof of contradiction for any real integer $k$, there can only be a finite amount of Pythagorean triples.






          share|cite|improve this answer












          Since the formula $a^2+b^2=c^2$ can be changed into $a^2=c^2-b^2$, $a^2$ needs to mean the difference between $2$ squares. You will find that $(a+1)^2-a^2$ is equal to $a^2-2a+1-a^2=2a+1$ and $(a+2)^2-a^2=a^2+4a+4-a^2=4a+4$. The former must be odd and the latter must be even. These are obtained by unpacking the bracket and simplifying. This is the maximum values of $c^2$ and $b^2$ of which $a^2=c^2-b^2$ stand true as if there were any larger then $c^2$ and $b^2$ are fractional. For $1$, the largest values of $c^2$ and $b^2$ is $0^2$ and $1^2$ respectively, but $0^2$ can't be part of a side. Again, $2^2$ is the difference between $0^2$ and $2^2$, but $0^2$ can't be used.



          On to the second question. We have already proven that the above formulae show that there is a limit as to the values of $b^2$ and $c^2$, and if there are infinite possible values of $b^2$ and $c^2$ then some of the must be non-integer. So by proof of contradiction for any real integer $k$, there can only be a finite amount of Pythagorean triples.







          share|cite|improve this answer












          share|cite|improve this answer



          share|cite|improve this answer










          answered Nov 17 at 2:14









          Kyky

          418213




          418213






















              up vote
              0
              down vote













              We have $(a^2+b^2)^2=(a^2-b^2)^2+(2ab)^2.$



              (1). The sequence $(2^2-1^2, 3^2-2^2, 4^2-3^2,...)$ of differences of successive squares is the sequence $(3,5,7,...)$ of odd numbers $>1$. So if $m $ is odd and $mgeq 3$ then we can find $b$ such that $(b+1)^2-b^2=m.$ That is, $2b+1=m.$ So let $b=(m-1)/2$ and $a=b+1.$ Then we have $$(a^2+b^2)^2=(a^2-b^2)+(2ab)^2=m^2+(2ab)^2.$$This does not work for $m=1$ as the member $2ab$ of the triplet would be $0.$



              (2). If $m$ is even and $mgeq 4$, let $a=m/2$ and $b=1.$ Then we have $$(a^2+b^2)^2 =(a^2-b^2)^2+(2ab)^2=(a^2-b^2)+m^2.$$This does not work for $m=2$ as the member $a^2-b^2$ of the triplet would be $0$.



              (3). If $x,y,z$ are positive integers with $x^2+y^2=z^2$ then



              (3-i). It is easy to confirm that $1ne zne 2.$



              (3-ii). We have $y<z$ so $ yleq z-1$ so $x^2=z^2-y^2geq z^2-(z-1)^2=2z-1geq 5$, so $x>2.$ Interchanging $x,y$ in this, we also get $y^2geq 5$ so $y>2$.



              BTW. The general formula for ALL Pyth. triplets is ${k(a^2-b^2),,2kab,, k(a^2+b^2)}$ for $a,b,kin Bbb N$ with $a>b$.






              share|cite|improve this answer

























                up vote
                0
                down vote













                We have $(a^2+b^2)^2=(a^2-b^2)^2+(2ab)^2.$



                (1). The sequence $(2^2-1^2, 3^2-2^2, 4^2-3^2,...)$ of differences of successive squares is the sequence $(3,5,7,...)$ of odd numbers $>1$. So if $m $ is odd and $mgeq 3$ then we can find $b$ such that $(b+1)^2-b^2=m.$ That is, $2b+1=m.$ So let $b=(m-1)/2$ and $a=b+1.$ Then we have $$(a^2+b^2)^2=(a^2-b^2)+(2ab)^2=m^2+(2ab)^2.$$This does not work for $m=1$ as the member $2ab$ of the triplet would be $0.$



                (2). If $m$ is even and $mgeq 4$, let $a=m/2$ and $b=1.$ Then we have $$(a^2+b^2)^2 =(a^2-b^2)^2+(2ab)^2=(a^2-b^2)+m^2.$$This does not work for $m=2$ as the member $a^2-b^2$ of the triplet would be $0$.



                (3). If $x,y,z$ are positive integers with $x^2+y^2=z^2$ then



                (3-i). It is easy to confirm that $1ne zne 2.$



                (3-ii). We have $y<z$ so $ yleq z-1$ so $x^2=z^2-y^2geq z^2-(z-1)^2=2z-1geq 5$, so $x>2.$ Interchanging $x,y$ in this, we also get $y^2geq 5$ so $y>2$.



                BTW. The general formula for ALL Pyth. triplets is ${k(a^2-b^2),,2kab,, k(a^2+b^2)}$ for $a,b,kin Bbb N$ with $a>b$.






                share|cite|improve this answer























                  up vote
                  0
                  down vote










                  up vote
                  0
                  down vote









                  We have $(a^2+b^2)^2=(a^2-b^2)^2+(2ab)^2.$



                  (1). The sequence $(2^2-1^2, 3^2-2^2, 4^2-3^2,...)$ of differences of successive squares is the sequence $(3,5,7,...)$ of odd numbers $>1$. So if $m $ is odd and $mgeq 3$ then we can find $b$ such that $(b+1)^2-b^2=m.$ That is, $2b+1=m.$ So let $b=(m-1)/2$ and $a=b+1.$ Then we have $$(a^2+b^2)^2=(a^2-b^2)+(2ab)^2=m^2+(2ab)^2.$$This does not work for $m=1$ as the member $2ab$ of the triplet would be $0.$



                  (2). If $m$ is even and $mgeq 4$, let $a=m/2$ and $b=1.$ Then we have $$(a^2+b^2)^2 =(a^2-b^2)^2+(2ab)^2=(a^2-b^2)+m^2.$$This does not work for $m=2$ as the member $a^2-b^2$ of the triplet would be $0$.



                  (3). If $x,y,z$ are positive integers with $x^2+y^2=z^2$ then



                  (3-i). It is easy to confirm that $1ne zne 2.$



                  (3-ii). We have $y<z$ so $ yleq z-1$ so $x^2=z^2-y^2geq z^2-(z-1)^2=2z-1geq 5$, so $x>2.$ Interchanging $x,y$ in this, we also get $y^2geq 5$ so $y>2$.



                  BTW. The general formula for ALL Pyth. triplets is ${k(a^2-b^2),,2kab,, k(a^2+b^2)}$ for $a,b,kin Bbb N$ with $a>b$.






                  share|cite|improve this answer












                  We have $(a^2+b^2)^2=(a^2-b^2)^2+(2ab)^2.$



                  (1). The sequence $(2^2-1^2, 3^2-2^2, 4^2-3^2,...)$ of differences of successive squares is the sequence $(3,5,7,...)$ of odd numbers $>1$. So if $m $ is odd and $mgeq 3$ then we can find $b$ such that $(b+1)^2-b^2=m.$ That is, $2b+1=m.$ So let $b=(m-1)/2$ and $a=b+1.$ Then we have $$(a^2+b^2)^2=(a^2-b^2)+(2ab)^2=m^2+(2ab)^2.$$This does not work for $m=1$ as the member $2ab$ of the triplet would be $0.$



                  (2). If $m$ is even and $mgeq 4$, let $a=m/2$ and $b=1.$ Then we have $$(a^2+b^2)^2 =(a^2-b^2)^2+(2ab)^2=(a^2-b^2)+m^2.$$This does not work for $m=2$ as the member $a^2-b^2$ of the triplet would be $0$.



                  (3). If $x,y,z$ are positive integers with $x^2+y^2=z^2$ then



                  (3-i). It is easy to confirm that $1ne zne 2.$



                  (3-ii). We have $y<z$ so $ yleq z-1$ so $x^2=z^2-y^2geq z^2-(z-1)^2=2z-1geq 5$, so $x>2.$ Interchanging $x,y$ in this, we also get $y^2geq 5$ so $y>2$.



                  BTW. The general formula for ALL Pyth. triplets is ${k(a^2-b^2),,2kab,, k(a^2+b^2)}$ for $a,b,kin Bbb N$ with $a>b$.







                  share|cite|improve this answer












                  share|cite|improve this answer



                  share|cite|improve this answer










                  answered Nov 17 at 13:38









                  DanielWainfleet

                  33.7k31647




                  33.7k31647






























                      draft saved

                      draft discarded




















































                      Thanks for contributing an answer to Mathematics Stack Exchange!


                      • Please be sure to answer the question. Provide details and share your research!

                      But avoid



                      • Asking for help, clarification, or responding to other answers.

                      • Making statements based on opinion; back them up with references or personal experience.


                      Use MathJax to format equations. MathJax reference.


                      To learn more, see our tips on writing great answers.





                      Some of your past answers have not been well-received, and you're in danger of being blocked from answering.


                      Please pay close attention to the following guidance:


                      • Please be sure to answer the question. Provide details and share your research!

                      But avoid



                      • Asking for help, clarification, or responding to other answers.

                      • Making statements based on opinion; back them up with references or personal experience.


                      To learn more, see our tips on writing great answers.




                      draft saved


                      draft discarded














                      StackExchange.ready(
                      function () {
                      StackExchange.openid.initPostLogin('.new-post-login', 'https%3a%2f%2fmath.stackexchange.com%2fquestions%2f3001873%2fpythagorean-triple%23new-answer', 'question_page');
                      }
                      );

                      Post as a guest















                      Required, but never shown





















































                      Required, but never shown














                      Required, but never shown












                      Required, but never shown







                      Required, but never shown

































                      Required, but never shown














                      Required, but never shown












                      Required, but never shown







                      Required, but never shown







                      Popular posts from this blog

                      Plaza Victoria

                      In PowerPoint, is there a keyboard shortcut for bulleted / numbered list?

                      How to put 3 figures in Latex with 2 figures side by side and 1 below these side by side images but in...